Tải bản đầy đủ (.pdf) (55 trang)

Tuyển tập các bài toán bất đẳng thức hay

Bạn đang xem bản rút gọn của tài liệu. Xem và tải ngay bản đầy đủ của tài liệu tại đây (639.2 KB, 55 trang )

Bài 1. Cho a, b ≥ 0. Chứng minh rằng

a
2
+ b +
3
4

b
2
+ a +
3
4



2a +
1
2

2b +
1
2

Lời giải.
Cách 1 . Không mấy khó khăn ta có thể dự đoán được bất đẳng thức này trở thành đẳng
thức khi a = b =
1
2
Khi đó theo bất đẳng thức AM-GM, ta có
a


2
+
1
4
≥ a, b
2
+
1
4
≥ b
Từ hai đánh giá này ta đưa bài toán về chứng minh bất đẳng thức mạnh hơn sau đây

a + b +
1
2

b + a +
1
2



2a +
1
2

2b +
1
2


Hay

a + b +
1
2

2


2a +
1
2

2b +
1
2

Nhưng điều này hiển nhiên đúng vì theo bất đẳng thức AM-GM, ta có

2a +
1
2

2b +
1
2






2a +
1
2
+ 2b +
1
2
2



2
=

a + b +
1
2

2
Bài toán được chứng minh xong.
Cách 2 . Bất đẳng thức cần chứng minh có thể viết lại như sau

a
2
+
1
2

+


b +
1
4

b
2
+
1
2

+

a +
1
4

≥ 4

a +
1
4

b +
1
4

Sử dụng bất đẳng thức AM-GM, ta có

a
2

+
1
2

+

b +
1
4

≥ 2


a
2
+
1
2

b +
1
4


b
2
+
1
2


+

a +
1
4

≥ 2


b
2
+
1
2

a +
1
4

Từ hai đánh giá này ta quy bài toán về chứng minh

a
2
+
1
2

b
2
+

1
2



a +
1
4

b +
1
4

Dễ thấy rằng bất đẳng thức này được suy ra từ đánh giá sau đây
x
2
+
1
2
≥ x +
1
4
Thế nhưng điều này là hiển nhiên đúng vì nó tưong đương với

x −
1
2

2
≥ 0

Bài toán được chứng minh xong.
50 Bai toan bat dang thuc
Trang 1
Bài 2. Cho các số dương x, y, z thỏa mãn x + y + z = 1. Chứng minh rằng
x
x +

x + yz
+
y
y +

y + zx
+
z
z +

z + xy
≤ 1
Lời giải.
Cách 1 . Trước hết, sử dụng giả thiết và bất đẳng thức Cauchy - Schwarz, ta có

x + yz =

x(x + y + z) + yz =

(x + y)(x + z) ≥ x +

yz
Do vậy

x
x +

x + yz

x
2x +

yz
Đến đây ta thiết lập thêm hai đánh giá tương tự để suy ra
x
x +

x + yz
+
y
y +

y + zx
+
z
z +

z + xy

x
2x +

yz
+

y
2y +

zx
+
z
2z +

xy
Do vậy, để kết thúc chứng minh, ta cần chỉ ra rằng
x
2x +

yz
+
y
2y +

zx
+
z
2z +

xy
≤ 1
Đặt a =

x, b =

y và c =


z, khi đó ta đưa bài toán về việc chứng minh
a
2
2a
2
+ bc
+
b
2
2b
2
+ ca
+
c
2
2c
2
+ ab
≤ 1
Bất đẳng thức này tương đương với dãy sau

1
2

a
2
2a
2
+ bc


+

1
2

b
2
2b
2
+ ca

+

1
2

c
2
2c
2
+ ab


3
2
− 1
bc
2a
2

+ bc
+
ca
2b
2
+ ca
+
ab
2c
2
+ ab
≥ 1
Đây là một đánh giá đúng vì theo bất đẳng thức Cauchy - Schwarz
bc
2a
2
+ bc
+
ca
2b
2
+ ca
+
ab
2c
2
+ ab
=
b
2

c
2
2a
2
bc + b
2
c
2
+
c
2
a
2
2b
2
ca + c
2
a
2
+
a
2
b
2
2c
2
ab + a
2
b
2


(ab + bc + ca)
2
a
2
b
2
+ b
2
c
2
+ c
2
a
2
+ 2abc(a + b + c)
= 1
Đẳng thức xảy ra khi và chỉ khi x = y = z =
1
3
.
Bài toán được chứng minh xong.
Cách 2 . Áp dụng bất đẳng thức Cauchy-Schwarz tương tự như trên nhưng bắt cặp khác
x +

x + yz = x +

x(x + y + z) + yz = x +

(x + z)(x + y) ≥ x +


xy +

xz
Từ đó ta có:
x
x +

x + yz

x
x +

xy +

xz
=

x

x +

y +

z
Làm tương tự và cộng các bất đẳng thức lại ta được điều phải chứng minh.
Bài toán được chứng minh xong.
50 Bai toan bat dang thuc
Trang 2
Bài 3. Cho a, b, c > 0. Chứng minh rằng

a(b + c)

bc(b
2
+ c
2
)
+
b(c + a)

ca(c
2
+ a
2
)
+
c(a + b)

ab(a
2
+ b
2
)
≥ 3

2
Lời giải.
Trước hết ta viết lại bất đẳng thức cần chứng minh như sau
a(b + c)


2bc(b
2
+ c
2
)
+
b(c + a)

2ca(c
2
+ a
2
)
+
c(a + b)

2ab(a
2
+ b
2
)
≥ 3
Ta dự đoán rằng đẳng thức xảy ra khi a = b = c. Khi đó ta có b
2
+ c
2
= 2bc. Áp dụng bất
đẳng thức AM-GM ta được

2bc(b

2
+ c
2
) ≤
2bc + b
2
+ c
2
2
=
(b + c)
2
2
Từ đó ta có
a(b + c)

2bc(b
2
+ c
2
)

2a
b + c
. Kết hợp với hai đánh giá tương tự khác, ta suy ra
a(b + c)

2bc(b
2
+ c

2
)
+
b(c + a)

2ca(c
2
+ a
2
)
+
c(a + b)

2ab(a
2
+ b
2
)
≥ 2

a
b + c
+
b
c + a
+
c
a + b

Cuối cùng, ta cần chỉ ra rằng

a
b + c
+
b
c + a
+
c
a + b

3
2
tuy nhiên đây lại là một đánh giá quen thuộc.
Đẳng thức xảy ra khi và chỉ khi a = b = c.
Bài toán được chứng minh xong. 
Bài 4. Cho a, b, c > 0 thoả mãn abc = 1. Chứng minh rằng
1
a
2
+
1
b
2
+
1
c
2
+ 3 ≥ 2(a + b + c)
Lời giải.
Do abc = 1 và a, b, c là các số thực dương nên ta có thể dự đoán được dấu bằng xảy ra khi
a = b = c = 1.

Dễ thấy rằng trong a, b, c tồn tại hai số cùng lớn hơn hoặc bằng 1, hoặc hai số đó cùng nhỏ
hơn 1.
Có thể thấy rõ hơn điều này thông qua chú ý sau

(a −1)(b −1)

(b −1)(c −1)

(c −a)(a −1)

= (a −1)
2
(b −1)
2
(c −1)
2
≥ 0
do đó trong ba số (a − 1)(b − 1), (b − 1)(c − 1), (c −1)(a − 1) có ít nhất một số không âm và
do tính đối xứng nên ta hoàn toàn có thể giả sử
(a −1)(b −1) ≥ 0
Không mất tính tổng quát, ta có thể giả sử hai số đó là a, b, suy ra
(a −1)(b −1) ≥ 0 ⇔ ab + 1 ≥ a + b ⇔ 2(ab + c + 1) ≥ 2(a + b + c)
50 Bai toan bat dang thuc
Trang 3
Ta sẽ chứng minh
1
a
2
+
1

b
2
+
1
c
2
+ 3 ≥ 2(ab + c + 1) ⇔
1
a
2
+
1
b
2
+
1
c
2
+ 1 ≥ 2(ab + c)
Đến đây, ta có thể sử dụng bất đẳng thức AM-GM cho hai số dương như sau
1
c
2
+ 1 ≥
2
c
=
2abc
c
= 2ab,

1
a
2
+
1
b
2

2
ab
=
2abc
ab
= 2c
Cộng hai bất đẳng thức trên lại ta suy ra điều phải chứng minh.
Đẳng thức xảy ra khi và chỉ khi a = b = c = 1.
Bài toán được chứng minh xong. 
Bài 5. Cho a ≥ b ≥ 1, a ≤ 3, ab ≤ 6, ab ≤ 6c. Chứng minh rằng
a + b −c ≤ 4
Lời giải.
Khi giải toán bất đẳng thức, lúc nào ta cũng nghĩ đến việc làm sao để sử dụng giả thiết cho
hiệu quả, bài này cũng không ngoại lệ.
Quan sát giả thiết của đề bài, các bạn có thấy chỉ có một giả thiết liên quan đến biến c thôi
không? Như vậy, để có thể chứng minh a + b − c ≤ 4 thì chắc chắn ta phải sử dụng giả thiết
này vào rồi. Lúc này, ta đưa được bài toán về chứng minh
a + b −
ab
6
≤ 4
Bây giờ, xem xét tiếp, ta thấy rằng giả thiết cho ta 3 ≥ a ≥ b ≥ 1, kết hợp với dự đoán dấu

bằng sẽ xảy ra khi a = 3, đồng thời ta cần có sự xuất hiện của a + b và ab trong các đánh giá
của mình (do bất đẳng thức cần chứng minh nó như vậy mà), ta nghĩ đến đánh giá sau đây
(3 −a)(3 −b) ≥ 0
Từ đánh giá này, ta suy ra 3(a + b) ≤ ab + 9, hay a + b ≤ 3 +
ab
3
. (Ta thực hiện đánh giá a + b
theo ab vì giả thiết bài toán thì có liên quan đến ab nhưng không có chút dính dáng nào tới
a + b cả).
Vậy là, ta chỉ cần chứng minh

3 +
ab
3


ab
6
≤ 4
Hay
ab ≤ 6
Cái này đúng do giả thiết!
Bài toán được chứng minh xong. 
Bài 6. Cho a ≥ b ≥ 1, a ≤ 3, ab ≤ 6, ab ≤ 6c. Chứng minh rằng
a + b −c ≤ 4
Lời giải.
50 Bai toan bat dang thuc
Trang 4
Cách 1 . Đối với các bất đẳng thức không thuần nhất thì cách đặt như trên tỏ ra rất hiệu quả
và rất hay được sử dụng. Mình cũng xin nêu ra một hướng tiếp cận khác cho bài toán này.

Mặc nhiên dễ thấy rằng đây là một bất đẳng thức hoán vị, mà như ta đã biết t hì bất đẳng
thức hoán vị thường khó xử lí hơn so với bất đẳng thức đối xứng vì thế ý tưởng của ta sẽ
tìm cách đưa bất đẳng thức này về dạng đối xứng. Hai công cụ để giúp ta thực hiện công
việc này chính là bất đẳng thức Cauchy-Schwarz và AM-GM.
Bất đẳng thức này hoán vị là do đại lượng ab
2
+ bc
2
+ ca
2
vì thế những đánh giá mà ta dùng
chắc hẳn phải liên quan đến đại lượngnày. Rất tựnhiêntheo bất đẳng thức Cauchy-Schwarz,
ta có
ab
2
+ bc
2
+ ca
2

(ab + bc + ca)
2
a + b + c
Từ đó đưa bài toán về chứng minh
a
2
+ b
2
+ c
2

+
(ab + bc + ca)
2
a + b + c
+ 9 ≥ 5(a + b + c)
Ta thấy rằng bất đẳng t hức này có chứa 3 đại lượng là a
2
+b
2
+c
2
, ab+bc+ca, a+b+c mà chúng
lại có liên quan với nhau thông qua hằng đẳng thức (a + b + c)
2
= a
2
+ b
2
+ c
2
+ 2(ab + bc + ca)
và nếu biểu diễn một đại lượng thông qua hai đại lượng còn lại thì bất đẳng thức của chúng
ta về cơ bản sẽ trở nên đơn giản hơn.
Với ý tưởng như vậy sử dụng hằng đẳng thức a
2
+ b
2
+ c
2
= (a + b + c)

2
−2(ab + bc + ca), ta
có thể viết bất đẳng thức chứng minh về dạng

(a + b + c)
2
− 6(a + b + c) + 9

+
(ab + bc + ca)
2
a + b + c
− 2(ab + bc + ca) + (a + b + c) ≥ 0
Hay
(a + b + c −3)
2
+
(ab + bc + ca −a −b −c)
2
a + b + c
≥ 0
Bất đẳng thức cuối cùng hiển nhiên đúng, tức bài toán đã được chứng minh xong.
Cách 2 . Đặt x = a −1, y = b−1, z = c −1, khi đó bất đẳng thức ban đầu được viết lại thành
(x+1)
2
+(y +1)
2
+(z +1)
2
+(x+1)(y +1)

2
+(y +1)(z +1)
2
+(z +1)(x+1)
2
+9 ≥ 5(x+y +z +3)
Hay (sau khi đã khai triển và rút gọn)
2(x
2
+ y
2
+ z
2
) + 2(xy + yz + zx) + xy
2
+ yz
2
+ zx
2
≥ 0
Lưu ý rằng từ phép đặt trên, ta suy ra x, y, z > −1, do đó đánh giá cuối cùng đúng vì
2(x
2
+y
2
+z
2
)+2(xy+yz+zx)+xy
2
+yz

2
+zx
2
= (x+y+z)
2
+y
2
(x+1)+z
2
(y+1)+x
2
(z+1) ≥ 0
Đẳng thức xảy ra khi và chỉ khi a = b = c = 1.
Bài toán được chứng minh xong.
Bài 7. Cho x, y, z > 0 thoả mãn x + y + z = 1. Tìm giá trị nhỏ nhất của
P =
x
2
(y + z)
yz
+
y
2
(z + x)
zx
+
z
2
(x + y)
xy

Lời giải.
50 Bai toan bat dang thuc
Trang 5
Cách 1 . Trước hết ta viết biểu thức bài cho lại như sau
P =

x
2
y
+
y
2
z
+
z
2
x

+

x
2
z
+
y
2
x
+
z
2

y

Tiếp đến ta sẽ chứng minh
x
2
y
+
y
2
z
+
z
2
x
≥ x + y + z (1)
Sử dụng AM-GM cho hai số
x
2
y
+ y ≥ 2x
rồi thực hiện tương tự chi hai đại lượng còn lại sau đó cộng vế lại với nhau ta sẽ được bất
đẳng thức (1).
Hoàn toàn tương tự cũng có
x
2
z
+
y
2
x

+
z
2
y
≥ x + y + z (2)
Cộng vế theo vế của (1) và (2), lại ta được
P ≥ 2(x + y + z) = 2
Vậy giá trị nhỏ nhất cần tìm là P = 2 đạt được khi và chỉ khi x = y = z =
1
3
.
Bài toán được chứng minh xong.
Cách 2 . Áp dụng bất đẳng thức Cauchy-Schwarz, ta có
P =
x
2
(y + z)
yz
+
y
2
(z + x)
zx
+
z
2
(x + y)
xy

(x + y + z)

2
yz
y + z
+
zx
z + x
+
xy
x + y
Theo một hệ quả trực tiếp của bất đẳng thức AM-GM, ta lại có
(y + z)
2
≥ 4yz
Nên
yz
y + z

y + z
4
Ta suy ra thêm hai đánh giá tương tự khác để có
P ≥
(x + y + z)
2
y + z
4
+
z + x
4
+
x + y

4
= 2(x + y + z)
Và vì x + y + z = 1 nên ta suy ra
P ≥ 2
Cuối cùng, với x = y = z =
1
3
(thoả mãn điều kiện) thì P = 2 nên ta kết luận 2 là giá trị nhỏ
nhất của biểu thức P.
Bài toán được chứng minh xong.
50 Bai toan bat dang thuc
Trang 6
Cách 3 . Dưới đây là một cách tiếp cận khác cho bài toán này.
Quan sát một chút ta t hấy rằng tử số của biểu thức bài cho có chứa các đại lượng x + y, y +
z, z + x còn ở mẫu số thì tương ứng chứa xy, yz, zx vì thế nếu ta sử dụng bất đẳng thức
AM-GM cho tử số thì có thể giản ước đi tử sô tuy nhiên lúc này bài toán lại xuất hiện căn
thức tuy nhiên các bạn đừng lo vì căn thức này nằm ở mẫu số vì thế ta có thể khử nó dễ
dàng bằng AM-GM.
Ý tưởng là vậy và ta tiến hành như sau, sử dụng bất đẳng thức AM-GM, ta có
x
2
(y + z)
yz

2x
2

yz
yz
=

2x
2

yz

4x
2
y + z
Thực hiện tượng tự cho hại đại lượng còn lại sau đó cộng tương ứng các vế lại với nhau ta
thu được
P ≥ 4

x
2
y + z
+
y
2
z + x
+
z
2
x + y

Mặt khác theo bất đẳng thức Cauchy-Schwarz thì
x
2
y + z
+
y

2
z + x
+
z
2
x + y

1
2
·
(x + y + z)
2
x + y + z
=
x + y + z
2
=
1
2
Từ đó suy ra P ≥ 2. Mặt khác dễ thấy với x = y = z =
1
3
thì P = 2. Việc tìm được các giá trị
của x, y, z thỏa mãn điều kiện bài toán và P = 2 cho phép ta kết luận giá trị nhỏ nhất của P
là 2.
Bài toán được giải quyết xong.
Ngoài ra ta có thể thu được đánh giá
x
2
y + z

+
y
2
z + x
+
z
2
x + y

x + y + z
2
Nhờ vào bất đẳng thức AM-GM như sau
x
2
y + z
+
y + z
4
≥ x
Bài toán được chứng minh xong.
Bài 8. Cho a, b, c > 0 và ab + bc + ca = a + b + c. Chứng minh rằng
a + b
a
2
+ b
2
+
b + c
b
2

+ c
2
+
c + a
c
2
+ a
2
≤ 3
Lời giải.
Cách 1 . Bài này vừa nhìn vào ta nghĩ ngay tới việc đánh giá cho các phân thức để đưa về
dạng đơn giản hơn. Mặt khác, sự xuất hiện của a + b và a
2
+ b
2
cùng với việc dấu bằng
xảy ra khi a = b = c = 1 làm ta không thể nào không nghĩ tới cái đánh giá quen thuộc
a
2
+ b
2

(a + b)
2
2
.
Ta có
a + b
a
2

+ b
2
+
b + c
b
2
+ c
2
+
c + a
c
2
+ a
2

2
a + b
+
2
b + c
+
2
c + a
Ta cần chứng minh
2
a + b
+
2
b + c
+

2
c + a
≤ 3
50 Bai toan bat dang thuc
Trang 7
Hay tương đương với
2
a + b
+
2
b + c
+
2
c + a

3(a + b + c)
ab + bc + ca
Ta nhân ab + bc + ca lên hai vế và để ý
ab + bc + ca
a + b
= c +
ab
a + b
, từ đó viết được bất đẳng
thức dưới dạng
2ab
a + b
+
2bc
b + c

+
2ca
c + a
≤ a + b + c
Sử dụng đánh giá quen thuộc
xy
x + y

x + y
4
và thu được ngay kết quả.
Bài toán được chứng minh xong.
Cách 2 . Ta có hai kết quả quen thuộc sau đây
(a + b)(b + c)(c + a) ≥
8
9
(a + b + c)(ab + bc + ca)
(a + b + c)
2
≥ 3(ab + bc + ca)
Trở lại bài toán sử dụng đánh giá quen thuộc 2(a
2
+ b
2
) ≥ (a + b)
2
ta đưa bài toán về chứng
minh
1
a + b

+
1
b + c
+
1
c + a

3
2
Hay dưới dạng đồng bậc là
1
a + b
+
1
b + c
+
1
c + a

3
2
a + b + c
ab + bc + ca
Bất đẳng thức này tương đương với
(a + b)(b + c) + (b + c)(c + a) + (c + a)(a + b)
(a + b)(b + c)(c + a)

3
2
a + b + c

ab + bc + ca
Hay
(a + b + c)
2
+ ab + bc + ca
(a + b)(b + c)(c + a)

3
2
a + b + c
ab + bc + ca
Sử dụng hai kết quả nêu ở trên, ta có
(a + b + c)
2
+ ab + bc + ca
(a + b)(b + c)(c + a)

(a + b + c)
2
+
(a + b + c)
2
3
8
9
(ab + bc + ca)(a + b + c)
=
3
2
a + b + c

ab + bc + ca
Bài toán được chứng minh xong.
Bài 9. Cho các số thực a, b, c ∈ [0, 1] thỏa mãn điều kiện a + b + c = 2. Tìm giá trị lớn nhất
của biểu thức sau
P =

a
2
− 4a + 5 +

b
2
− 4b + 5 +

c
2
− 4c + 5
Lời giải.
Thứ nhất, dự đoán dấu bằng để đạt giá trị lớn nhất. Không khó để ta có thể dự đoán một
cách "tự nhiên" là khi a = b = 1, c = 0 thì biểu thức đạt giá trị lớn nhất.
50 Bai toan bat dang thuc
Trang 8
Thứ hai, "liều" với dự đoán ở trên, ta tìm cách đánh giá cho hiệu quả.
Quan sát thấy biểu thức của ta là tổng đối xứng ba biến và hơn nữa nó có thể viết lại dưới
dạng
P = f(a) + f(b) + f(c)
với f (x) =

x
2

− 4x + 5. Như vậy, một cách tự nhiên, ta mong một đánh giá "riêng lẻ" hay
"chia để trị" kiểu
f(x) =

x
2
− 4x + 5 ≤ mx + n,
đúng với x ∈ [0, 1] với m, n là các số thực nào đó.
Để khi đó, ta có thể có
P = f(a) + f(b) + f(c) ≤ m(a + b + c) + 3n = 2n + 3m
nói một cách khác, ta đã tìm được một chặn trên của P mà ta mong đó là giá trị lớn nhất.
Công việc của ta là đi tìm m, n.
Như đã nói ở trên, ta cần đánh giá để đảm bảo dấu bằng, mà vì các biến là đối xứng với
nhau nên ta không thể quyết định biến nào bằng 0 hay biến nào bằng 1 khi biểu thức P đạt
giá trị lớn nhất. Bởi vậy, tốt nhất, ta sẽ chọn m, n sao cho mà dấu bằng tại 0 và 1 đều thỏa.
Nói một cách khác, ta cần tìm m, n sao cho phương trình

x
2
− 4x + 5 −(mx + n) = 0
có hai nghiệm là 0 và 1.
Thay x = 0 và x = 1, ta tìm được m =

2 −

5 và n =

5. Như vậy, ta mong là bất đẳng
thức sau đúng


x
2
− 4x + 5 ≤


2 −

5

x +

5
Nhưng kì diệu thay, bất đẳng thức này hoàn toàn đúng với điều kiên x ∈ [0, 1] (công việc
này hết sức đơn giản, và thực ra ta chỉ cần lý luận thôi cũng đã có thể đảm bảo tính đúng
đắn của bất đẳng thức này, xin dành lại cho mọi người).
Như vậy, ta sẽ có
P = f(a) + f(b) + f(c) ≤


2 −

5

(a + b + c) + 3

5 = 2

2 +

5

Với a = b = 1, c = 0 ta có ngay GTNN của P = 2

2 +

5.
Bài toán được chứng minh xong. 
Bài 10. Cho a, b, c > 0 và a + b + c = abc. Chứng minh rằng
a
b
3
+
b
c
3
+
c
a
3
≥ 1
Lời giải.
Cách 1 . Bất đẳng thức này không đồng bậc, nhận thấy vế trái bậc (−2), vế phải bậc 0, điều
kiện giả thiết a + b + c (bậc 1) = abc (bậc 3), từ đó ta có ý tưởng làm cho hai biểu thức đồng
bậc. Với ý tưởng như vậy ta sẽ tiến hành như sau.
Bất đẳng thức cần chứng minh tương đương với
abc

a
b
3
+

b
c
3
+
c
a
3

≥ a + b + c
Hay
a
2
c
b
2
+
b
2
a
c
2
+
c
2
b
a
2
≥ a + b + c
50 Bai toan bat dang thuc
Trang 9

Đến đây sử dụng bất đẳng thức Cauchy-Schwarz, ta có

1
c
+
1
a
+
1
b

a
2
c
b
2
+
b
2
a
c
2
+
c
2
b
a
2




a
b
+
b
c
+
c
a

2
Ta cần chứng minh

a
b
+
b
c
+
c
a

2
≥ (a + b + c)

1
a
+
1
b

+
1
c

Hay
a
2
b
2
+
b
2
c
2
+
c
2
a
2
+
a
c
+
b
a
+
c
b
≥ 3 +
a

b
+
b
c
+
c
a
Theo bất đẳng thức AM-GM, ta có

a
2
b
2
+ 1

+

b
2
c
2
+ 1

+

c
2
a
2
+ 1


≥ 2

a
b
+
b
c
+
c
a


a
b
+
b
c
+
c
a
+
a
c
+
c
b
+
b
a

≥ 6
Cộng lại, ta có điều phải chứng minh và đẳng thức xảy ra khi và chỉ khi a = b = c =

3.
Bài toán được chứng minh xong.
Cách 2 . Một lời giải khá tự nhiên cho bài này là dùng cách đặt ẩn phụ để đưa điều kiện bài
toán về dạng dễ nhìn.
Đặt
1
a
= x,
1
b
= y,
1
c
= z, ta suy ra xy + yz + zx = 1. Khi đó
a
b
3
+
b
c
3
+
c
a
3
=
y

3
x
+
z
3
y
+
x
3
z
Ta chỉ cần chứng minh
y
3
x
+
z
3
y
+
x
3
z
≥ 1
điều này khá dễ, xin không chứng minh.
Bài toán được chứng minh xong.
Cách 3 . Ta viết bất đẳng thức cần chứng minh lại dưới dạng thuần nhất như sau
a
b
3
+

b
c
3
+
c
a
3

a + b + c
abc
Nhân hai vế của bất đẳng thức này với abc > 0, ta được
ab
2
c
2
+
bc
2
a
2
+
ca
2
b
2
≥ a + b + c
Sử dụng bất đẳng thức AM-GM, ta có
bc
2
a

2
+
ca
2
b
2
+ b ≥ 3
3

bc
2
a
2
·
ca
2
b
2
· b = 3c
Thực hiện đánh giá tương tự cho các đại lượng còn lại sau đó cộng tương ứng các vế lại với
nhau ta thu được kết quả cần phải chứng minh.
Bài toán được chứng minh xong.
50 Bai toan bat dang thuc
Trang 10
Bài 11. Khi hệ

x
2
+ xy + y
2

= 3
y
2
+ yz + z
2
= 16
có nghiệm. Chứng minh rằng
xy + yz + zx ≤ 8
Lời giải.
Cách 1 . Sử dụng bất đẳng thức Cauchy-Schwarz, ta có
(xy + yz + zx)
2
=

x

y +
z
2

+ z

y +
z
2

2


x

2
+
4
3

y +
x
2

2


y +
z
2

2
+
3
4
z
2

=
4
3
(x
2
+ xy + y
2

)(y
2
+ yz + z
2
) = 64
Từ đó ta dễ dàng suy ra ra được
xy + yz + zx ≤ 8
Với đẳng thức xảy ra khi và chỉ
x =
7

31
, y =
4

31
, z =
20

31
Bài toán được chứng minh xong.
Cách 2 . Để ý rằng
48 = (x
2
+ xy + y
2
)(y
2
+ yz + z
2

) =



y +
x
2

2
+

x

3
2

2





z

3
2

2
+


y +
z
2

2


Áp dụng bất đẳng thức Cauchy-Schwarz ta có:
48 ≥

yz

3
2
+
xz

3
4
+
xy

3
2
+
xz

3
4


2
=

(xy + yz + zx)

3
2

2
Từ đó suy ra xy + yz + zx ≤ 8.
Bài toán được chứng minh xong.
Cách 3 . Ta thấy các biểu thức xy + yz + zx, x
2
+ xy + y
2
, y
2
+ yz + z
2
đều có bậc hai, thế nên
khi đồng bậc hóa ta sẽ phải đưa bất đẳng thức về dạng
xy + yz + zx ≤ m(x
2
+ xy + y
2
) + n(y
2
+ yz + z
2
) (1)

Bây giờ, ta cần chứng minh xy + yz + zx ≤ 8 nên tất nhiên ta bắt buộc phải chọn m, n sao
cho m(x
2
+ xy + y
2
) + n(y
2
+ yz + z
2
) = 8. Với điều kiện của bài toán thì ta có điều này khi
3m + 16n = 8
Ra được một phương trình cho m, n, ta tìm thêm một phương trình nữa là xong.
Ta viết lại (1) dưới dạng
f(x) = mx
2
+ [(m −1)y − z] x + (m + n)y
2
+ (n −1)yz + nz
2
≥ 0
Như đã nói ở trên, ta mong muốn f(x) ≥ 0, ∀x ∈ R. Vậy thì với phần kiến thức "lượm" được
ở trên, có phải là ta phải chọn m, n sao cho m > 0 và
[(m −1)y − z]
2
≤ 4m

(m + n)y
2
+ (n −1)yz + nz
2


, ∀y, z ∈ R (2)
50 Bai toan bat dang thuc
Trang 11
không các bạn? Tiếp tục, ta viết lại (2) dưới dạng
g(z) = (4mn −1)z
2
+ 2(2mn −m −1)yz + (3m
2
+ 4mn + 2m −1)y
2
≥ 0
Ở đây, ta chọn xét tam thức theo z sẽ gọn gàng hơn! Rõ ràng để g(z) ≥ 0, ∀z ∈ R thì ta phải
có 4mn > 1 và


= (2mn −m −1)
2
y
2
− (4mn −1)(3m
2
+ 4mn + 2m −1)y
2
≤ 0
Ngoài ra, do đề bài yêu cầu chứng minh ≤ chứ không phải < nên khả năng đẳng thức có thể
xảy ra là rất cao (dù ta chưa biết được tại đâu). Vì vậy, tốt nhất là ta nên chọn m, n sao cho
bất đẳng thức (1) có thể xảy ra dấu bằng. Muốn vậy thì ta phải chọn m, n sao cho ∆

g

= 0 với
mọi y ∈ R (tại vì chỉ có vậy thì ∆
f
mới có t hể bằng 0 và đẳng thức mới có thể xảy ra). Chú ý
rằng ∆
f
luôn bằng 0 khi y = z = 0 nhưng tại các điểm này thì điều kiện y
2
+ yz + z
2
= 16
không được thỏa mãn, vì vậy mà ta phải chọn như trên. Cuối cùng ta thu được hệ









3m + 16n = 8
m > 0
4mn > 1
(2mn −m −1)
2
− (4mn −1)(3m
2
+ 4mn + 2m −1) = 0
Giải hệ này, ta được m =

4
3
, n =
1
3
, sau đó ráp vào bất đẳng thức ban đầu ta được điều phải
chứng minh.
Bài toán được chứng minh xong.
Bài 12. Cho a, b, c > 0. Chứng minh rằng
ab
2
c
+
bc
2
a
+
ca
2
b
+ a
2
+ b
2
+ c
2
≥ 2

(ab
3

+ bc
3
+ ca
3
)

a
b
+
b
c
+
c
a

Lời giải.
Với bất đẳng thức chứa căn, điều đầu tiên ta phải làm đó là tìm cách phá căn. Trong bài toán
này, để làm được điều đó, ta liên tưởng ngay tới bất đẳng thức AM-GM dạng 2

xy ≤ x + y.
Nguyên tắc tiếp theo, khi sử dụng AM-GM, cần phải đảm bảo điều kiện đồng bậc của x và
y, đó là lí do ta không nên đánh giá như sau:
2

(ab
3
+ bc
3
+ ca
3

)

a
b
+
b
c
+
c
a

≤ ab
3
+ bc
3
+ ca
3
+
a
b
+
b
c
+
c
a
(Bởi một ông bậc 4 và một ông bậc 0, chẳng liên quan gì đến nhau cả).
Mặt khác, để ý rằng
ab
2

c
=
ab
3
bc
. Từ đó, có lời giải sau.
Đây là một bất đẳng thức hoán vị nên không mất tính tổng quát, ta hoàn toàn có thể giả sử
a là số nằm giữa a, b, c.
Sử dụng bất đẳng thức AM-GM dạng 2

xy ≤ x + y, ta có
2

(ab
3
+ bc
3
+ ca
3
)

a
b
+
b
c
+
c
a



ab
3
+ bc
3
+ ca
3
bc
+ bc

a
b
+
b
c
+
c
a

Vậy chứng minh sẽ hoàn tất nếu ta chỉ ra được rằng
ab
3
+ bc
3
+ ca
3
bc
+ bc

a

b
+
b
c
+
c
a


ab
2
c
+
bc
2
a
+
ca
2
b
+ a
2
+ b
2
+ c
2
50 Bai toan bat dang thuc
Trang 12
Hay
ab

2
c
+ c
2
+
a
3
b
+ ca + b
2
+
bc
2
a

ab
2
c
+
bc
2
a
+
ca
2
b
+ a
2
+ b
2

+ c
2
Tương đương với
a(a −b)(a −c)
b
≤ 0
Hiển nhiên đúng do giả sử a là số nằm giữa a, b, c.
Dấu bằng xảy ra khi và chỉ khi a = b = c > 0
Bài toán được chứng minh xong. 
Bài 13. Cho x, y, z ∈ [0, 1] thỏa mãn
1
4x + 5
+
2
4y + 5
+
3
4z + 5
= 1. Tìm giá trị lớn nhất của
P = xy
2
z
3
Lời giải.
Cách 1 . Ta chứng minh hai bổ để sau
Bổ đề 1.Với x, y ∈ [0, 1] thì
1
4x + 5
+
1

4y + 5

2
4

xy + 5
Chứng minh
Quy đồng mẫu số và rút gọn, ta thu được (20 −16

xy)(

x −

y)
2
≥ 0, hiển nhiên đúng với
x, y ∈ [0, 1]
Bổ đề 2. Với x, y, z ∈ [0, 1]thì
1
4x + 5
+
1
4y + 5
+
1
4z + 5

3
4
3


xyz + 5
Chứng minh
Cộng thêm cả hai vế với
1
4
3

xyz + 5
và áp dụng liên tiếp hai lần bổ đề 1.
Quay trở lại bài toán, sử dụng liên tiếp hai bổ đề trên, ta dễ dàng chứng minh được:
Với 6 số thực a
1
, a
2
, a
3
, a
4
, a
5
, a
6
∈ [0, 1], ta có
1
4a
1
+ 5
+
1

4a
2
+ 5
+
1
4a
3
+ 5
+
1
4a
4
+ 5
+
1
4a
5
+ 5
+
1
4a
6
+ 5

6
4
6

a
1

.a
2
.a
3
.a
4
.a
5
.a
6
+ 5
Tới đây, lấy a
1
= x, a
2
= a
3
= y, a
4
= a
5
= a
6
= z thì bất đẳng thức trên trở thành
1
4x + 5
+
2
4y + 5
+

3
4z + 5

6
4
6

xy
2
z
3
+ 5
Kết hợp với giả thiết, ta thu được
1 ≤
6
4
6

xy
2
z
3
+ 5
Từ đó dễ dàng suy ra P = xy
2
z
3

1
4

6
Bài toán được chứng minh xong.
50 Bai toan bat dang thuc
Trang 13
Cách 2 . Viết giải thiết thành
1
4x + 5
+
1
4y + 5
+
1
4y + 5
+
1
4z + 5
+
1
4z + 5
+
1
4z + 5
= 1
Ta có
1
4x + 5
=
1
5


1
4y + 5
+
1
5

1
4y + 5
+
1
5

1
4z + 5
+
1
5

1
4z + 5
+
1
5

1
4z + 5
=
1
5


4y
4y + 5

+
1
5

4y
4y + 5

+
1
5

4z
4z + 5

+
1
5

4z
4z + 5

+
1
5

4z
4z + 5


Áp dụng bất đẳng thức AM − GM, ta có
1
4x + 5
≥ 4
5

y
2
z
3
(4y + 5)
2
(4z + 5)
3
(1)
Tương tự ta có
1
4y + 5
≥ 4
5

xyz
3
(4x + 5)(4y + 5)(4z + 5)
3
(2)
1
4z + 5
≥ 4

5

xyz
3
(4x + 5)(4y + 5)
2
(4z + 5)
2
(3)
Bình phương hai vế của bất đẳng thức (2) và lập phương hai vế của bất đẳng thức (3) rồi
nhân vế theo vế của (1), (2), (3) , ta được
1 ≥ 4
6
xy
2
z
3
Do đó,
xy
2
z
3

1
4
6
Đẳng thức xảy ra khi và chỉ khi
x = y = z =
1
4

Vậy giá trị lớn nhất của P bằng
1
4
6
Bài toán được chứng minh xong.
Bài 14. Cho 4 số thực a, b, c, d thỏa mãn: abcd > a
2
+ b
2
+ c
2
+ d
2
. Chứng minh rằng
abcd > a + b + c + d + 8
Lời giải.
Áp dụng bất đẳng thức AM-GM ta có
abcd > a
2
+ b
2
+ c
2
+ d
2
≥ 4
2

abcd ⇔ abcd > 16
Áp dụng bất đẳng thức Cauchy-Schwarz ta có

abcd > a
2
+ b
2
+ c
2
+ d
2

(a + b + c + d)
2
4
Đặt k = a + b + c + d nên ta cần chứng minh
k
2
− 4k − 32 > 0 ⇔ (k − 8)(k + 4) > 0
50 Bai toan bat dang thuc
Trang 14
Nếu a + b + c + d < 8 suy ra a + b + c + d + 8 < 16 < abcd ta có điều phải chứng minh.
Nếu a + b + c + d ≥ 8 và cũng suy ra điều phải chứng minh.
Bài toán được chứng minh xong. 
Bài 15. Cho x, y, z là các số thực thỏa mãn x
2
+ y
2
+ z
2
= 3. Tìm giá trị lớn nhất của
P = xy + yz + 2zx
Lời giải.

Cách 1 . Ta chỉ cần xét trường hợp x, y, z > 0
Từ x
2
+ y
2
+ z
2
= 3 nên bao giờ cũng có thể đặt y =

3 cos a, x =

3 sin a cos b, z =

3 sin a sin b. Với a, b ∈ [0;
π
2
]
Thế thì
P = 3

2 sin a cos a sin(b +
π
4
) + 3 sin
2
a sin 2b
Ta có đánh giá sau
P ≤
3


2
2
sin 2a + 3 sin
2
a =
3
2
+
3
2


2 sin 2a − cos 2a


3
2
(1 +

3)
Đẳng thức xảy ra khi và chỉ khi







y =


3 −

3
2
x = z =

3 +

3
4
Vậy giá trị lớn nhất của P bằng
3
2

1 +

3

.
Bài toán được chứng minh xong.
Cách 2 . Đặt a
2
=

3 −1
2
và 2b
2
= a
2

+ 1 (với a, b > 0). Khi đó ta có ab =
1
2
. Do đó ta có các
đánh giá sau đây
a
2
x
2
+ b
2
y
2
≥ 2abxy
b
2
y
2
+ a
2
z
2
≥ 2abyz
x
2
+ z
2
≥ 2xz
Suy ra
(a

2
+ 1)x
2
+ 2b
2
y
2
+ (a
2
+ 1)z
2
≥ (2abxy + 2abyz + 2xz)
Hay
2b
2
(x
2
+ y
2
+ z
2
) ≥ (xy + yz + 2xz)
Bài toán được chứng minh xong.
Cách 3 . Trước tiên, ta sẽ đồng bậc hóa bất đẳng thức đã cho để đưa bài toán về đúng bản
chất của nó. Bất đẳng thức đồng bậc của ta có dạng
xy + yz + 2zx ≤ k(x
2
+ y
2
+ z

2
) (∗)
Với hằng số k > 0.
Trở lại bài toán cần xét, khi đọc tới giả thiết x
2
+y
2
+z
2
= 3 thì phải hiểu rằng Max P = 3k và
công việc của ta là tìm k để (*) luôn đúng với mọi x, y, z. Để ý rằng, (*) có thể viết lại thành
ky
2
− y(x + z) + kx
2
+ kz
2
− 2xz ≥ 0
50 Bai toan bat dang thuc
Trang 15
Do hệ số k > 0 nên coi đây là tam thức bậc hai ẩn y thì dễ thấy nếu ta tìm được k sao cho
∆ ≤ 0, ∀x, z, bài toán sẽ được giải quyết (theo định lí về dấu của tam thức bậc hai).
Ta lại có ∆ = (1 −4k
2
)(x
2
+ z
2
) + 2(1 + 4k)xz (1)
Mặt khác, bất đẳng thức trên đối xứng với x và z nên ta có thể dự đoán P đạt Max khi và chỉ

khi x = z. Từ đó, dẫn tới ý tưởng thay x = z = 1 vào (1), ta thu được 2k
2
− 2k − 1 = 0.
Do k > 0 nên ta chỉ chọn nghiệm k =
1 +

3
2
mà thôi.
Vậy kết luận được Max P =
3(1 +

3)
2
Bài toán được chứng minh xong.
Cách 4 . Ta chỉ cần giải bài toán trong trường hợp a, b, c là các số dương là được.
Giả sử rằng P sẽ đạt giá trị lớn nhất khi a = x, b = y, c = z khi đó dễ thấy
a
x
=
b
y
=
c
z
= 1
và x
2
+ y
2

+ z
2
= 3. Khi đó theo bất đẳng thức AM-GM, ta có
ab =
1
2xy
· 2 · ay ·bx ≤
1
2xy

a
2
y
2
+ b
2
x
2

= a
2
·
y
2x
+ b
2
·
x
2y
Đánh giá tương tự cho bc và 2ca ta được

bc ≤ b
2
·
z
2y
+ c
2
·
y
2z
2ca ≤ c
2
·
x
z
+ a
2
·
z
x
Cộng tương ứng các vế của ba bất đẳng thức trên, ta thu được
P = ab + bc + 2ca ≤ a
2
·

y + 2z
2x

+ b
2

·

z + x
2y

+ c
2
·

2x + y
2z

Vậy ta chỉ cần chọn x, y, z sao cho



y + 2z
x
=
z + x
y
=
2x + y
z
x
2
+ y
2
+ z
2

= 3
Ta có phương trình
y + 2z
x
=
2x + y
z
tương đương với (z − x)[y + 2(z + x)] = 0 tức x = z (vì x, y, z > 0). Thay giá trị này vào
phương trình
y + 2z
x
=
z + x
y
ta sẽ thu được
2x
2
− 2xy − y
2
= 0
hay
2

x
y

2
− 2 ·
x
y

− 1 = 0
Giải phương trình này tathu được nghiệm là
x
y
=
1 +

3
2
. Từ đó ta thu được x = y·
1 +

3
2
=
z từ đây kết hợp với giả thiết x
2
+ y
2
+ z
2
= 3 ta tìm được












x = z =

3 +

3
4
y =

3 −

3
2
50 Bai toan bat dang thuc
Trang 16
và từ đó tìm được giá trị lớn nhất của P là
3
2

1 +

3

.
Thực ra ta có thể dựa vào tính đối xứng của c, a để dự đoán đẳng t hức xảy ra khi c = a =
x, z = y để đưa ra phân tích ngắn hơn nhưng mình muốn trình bày một cách tổng quan hơn
nên đã chọn phương pháp trên.
Bài toán được chứng minh xong.

Bài 16. Cho x, y, z > 0 và xyz = 8. Tìm giá trị lớn nhất của biểu thức
P =
1
2x + y + 6
+
1
2y + z + 6
+
1
2z + x + 6
Lời giải.
Cách 1 . Đầu tiên hãy quan sát điều kiện bài toán "xyz = 8" ??!! Nhìn có vẻ hơi lạ mắt vì ta
thường biết đến với giả thiết quen thuộc là xyz = 1 vì thế ý nghĩ ngay lúc này của ta là tìm
cách đưa giả thiết này về dạng xyz = 1 và vì thế phép đặt x = 2a, y = 2b, z = 2c cũng rất tự
nhiên được chúng ta nghĩ đến. Ngoài ra với phép đặt này thì
2x + y + 6 = 2(2a + b + 3), 2y + z + 6 = 2(2b + c + 3), 2z + x + 6 = 2(2c + a + 3).
Lúc này
P =
1
2

1
2a + b + 3
+
1
2b + c + 3
+
1
2c + a + 3


Rõ ràng thì P lúc này nhìn có vẻ đơn giản hơn sao với P ban đầu. Tiếp đến là việc điểm rơi
của bài toán, biểu thức bài cho là đối xứng vì thế ta có thể dự đoán được nó sẽ lớn nhất khi
các biến bằng nhau, mà cụ thể trong bài toán này thì a = b = c = 1. Việc còn lại là ta sẽ chọn
công cụ để chứng minh bài toán. Thật đơn giản hình thức phát biểu của P lại giống hệt như
bất đẳng thức Cauchy-Schwarz (hoặc bất đẳng thức AM-GM dạng cộng mẫu) là
1
x
1
+
1
x
2
+ ··· +
1
x
n

n
2
x
1
+ x
2
+ ··· + x
n
Vậy "thanh đao" mà ta dùng để "trảm" bài toán này nếu không là Cauchy-Schwarz thì là
AM-GM.
Vì điểm rơi mà ta dự đoán là a = b = c = 1 nên những đánh giá mà ta dùng cũng phải đảm
bảo điều này thì mới có hi vọng giải quyết được bài toán và công việc cần làm của ta là chọn
các biến x

1
, x
2
, . . . sao cho chúng bằng nhau. Dễ thấy rằng nếu a = b = c = 1 thì
2a + b + 3 = 2b + c + 3 = 2c + a + 3 = 6
mà 6 chỉ có thể phân tích thành tổng các số bằng nhau như sau
6 = 1 + 1 + 1 + 1 + 1 + 1 = 2 + 2 + 2 = 3 + 3
Và ta sẽ chọn phương án phân tích cuối cùng tức là tách 6 = 3+3 vì hai cách còn lại sẽ không
giải quyết được bài toán (các bạn có thể tự kiểm tra) tuy nhiên ta lại cũng có hai cách phân
tích
2a + b + 3 = (a + b + 1) + (a + 2) = (2a + 1) + (b + 2)
và mình sẽ chọn cách phân tích thứ nhất (vì sao lại không chọn cách hai?).
Biểu thức đầu bài là đối xứng điều này cho phép ta dự đoán nó sẽ đạt giá trị lớn nhất khi
a = b = c = 1 (vì thường thì là như vậy mà). Từ đó dự đoán rằng bất đẳng thức sau đây sẽ
đúng
1
a + 2b + 3
+
1
b + 2c + 3
+
1
c + 2a + 3

1
2
50 Bai toan bat dang thuc
Trang 17
Bây giờ ta sẽ đi kiểm chứng dự đoán của mình. Sử dụng bất đẳng thức Cauchy-Schwarz, ta


1
a + 2b + 3
=
1
(a + b + 1) + (b + 2)

1
4

1
a + b + 1
+
1
b + 2

Đánh giá tương tự cho hai đại lượng còn lại và cộng tương ứng các vế lại với nhau ta thu
được
P ≤
1
4

1
a + b + 1
+
1
b + c + 1
+
1
c + a + 1
+

1
a + 2
+
1
b + 2
+
1
c + 2

Vậy ta sẽ đi chứng minh
1
a + b + 1
+
1
b + c + 1
+
1
c + a + 1
+
1
a + 2
+
1
b + 2
+
1
c + 2
≤ 2
Bất đẳng thức này có được bằng cách cộng hai bất đẳng thức sau đây
1

a + b + 1
+
1
b + c + 1
+
1
c + a + 1
≤ 1
1
a + 2
+
1
b + 2
+
1
c + 2
≤ 1
Ta sẽ chứng minh bất đẳng thức thứ nhất, đặt a = x
2
, b = y
2
, c = z
2
thì xyz ≥ 1 và ta cần
chứng minh
1
x
2
+ y
2

+ 1
+
1
y
2
+ z
2
+ 1
+
1
z
2
+ x
2
+ 1
≤ 1
Theo bất đẳng thức Cauchy-Schwarz, ta có
1
x
2
+ y
2
+ 1
=
z
2
+ 2
(x
2
+ y

2
+ 1)(1 + 1 + z
2
)

z
2
+ 2
(x + y + z)
2
Đánh giá tượng tự cho hai số hạng còn lại ta thu được
1
x
2
+ y
2
+ 1
+
1
y
2
+ z
2
+ 1
+
1
z
2
+ x
2

+ 1

x
2
+ y
2
+ z
2
+ 6
(x + y + z)
2
Vậy ta sẽ chứng minh
(x + y + z)
2
≥ x
2
+ y
2
+ z
2
+ 6
hay là
xy + yz + zx ≥ 3
thế nhưng bất đẳng thức này hiển nhiên đúng theo bất đẳng thức AM-GM cho ba số.
Tiếp đến là chứng minh bất đẳng thức thức thứ hai. Ta cũng đổi biến tương tự như trên để
đưa bất đẳng thức về chứng minh
1
x
2
+ 2

+
1
y
2
+ 2
+
1
z
2
+ 2
≤ 1
với điều kiện xyz ≥ 1. Bất đẳng này tương đương với

1
2

1
x
2
+ 2

+

1
2

1
y
2
+ 2


+

1
2

1
z
2
+ 2


3
2
− 1
hay là
x
2
x
2
+ 2
+
y
2
y
2
+ 2
+
z
2

z
2
+ 2
≥ 1
50 Bai toan bat dang thuc
Trang 18
Sử dụng bất đẳng thức Cauchy-Schwarz, ta có
x
2
x
2
+ 2
+
y
2
y
2
+ 2
+
z
2
z
2
+ 2

(x + y + z)
2
x
2
+ y

2
+ z
2
+ 6
Cuối cùng ta sẽ chứng minh
(x + y + z)
2
x
2
+ y
2
+ z
2
+ 6
≥ 1
hiển nhiên đúng theo kết quả ở trên. Vậy dự đoán của ta là chính xác nên giá trị lớn nhất của
P là
1
2
đạt được khi a = b = c = 1.
Bài toán được chứng minh xong.
Cách 2 . Đặt a =
x
2
, b =
y
2
, c =
z
2

. Khi đó ta có abc = 1 và
P =
1
2

1
2a + b + 3
+
1
2b + c + 3
+
1
2c + a + 3


1
8

1
a + b + 1
+
1
b + c + 1
+
1
c + a + 1
+
1
a + 2
+

1
b + 2
+
1
c + 2

Ta có
1
a + b + 1

1
3

ab

3

a +
3

b

+ 1
=
1
1
3

c


3

a +
3

b

+ 1
=
3

c
3

a +
3

b +
3

c
Khi đó ta có
A =
1
a + b + 1
+
1
b + c + 1
+
1

c + a + 1
≤ 1
Ta chứng minh
B =
1
a + 2
+
1
b + 2
+
1
c + 2
≤ 1
Bất đẳng thức này khi quy đồng lên ta được
ab + bc + ca ≥ 3
Bất đẳng thức cuối luôn đúng, do đó ta có P ≤
1
4
.Đẳng thức xảy ra khi x = y = z = 2.
Bài toán được chứng minh xong.
Cách 3 . Đặt
x
2
= a
2
,
y
2
= b
2

,
z
2
= c
2
với a, b, c > 0 thì ta cũng có abc = 1. Biểu t hức trở thành
2P =
1
2a
2
+ b
2
+ 3
+
1
2b
2
+ c
2
+ 3
+
1
2c
2
+ a
2
+ 3
Áp dụng bất đẳng thức AM-GM, ta có
2P ≤
1

2a + 2ab + 2
+
1
2b + 2bc + 2
+
1
2c + 2ca + 2
=
1
2
⇒ P ≤
1
4
Bài toán được chứng minh xong.
50 Bai toan bat dang thuc
Trang 19
Bài 17. Cho x, y = 0, xy(x + y) = x
2
+ y
2
− xy. Tìm giá trị lớn nhất của
P =
1
x
3
+
1
y
3
Lời giải.

Ta có
xy(x + y) = x
2
+ y
2
− xy ⇔
1
x
+
1
y
=
1
x
2
+
1
y
2
− 1
Đặt a =
1
x
; b =
1
y
. Điều kiện bài toán trở thành
a + b = a
2
+ b

2
− ab
Và ta cần tìm GTLN của
P = a
3
+ b
3
Ta có
P = a
3
+ b
3
= (a + b)
3
− 3ab(a + b) = (a + b)(a
2
+ b
2
− ab) = (a + b)
2
Ta có
a + b = (a + b)
2
− 3ab ≥ (a + b)
2

3(a + b)
2
4
=

(a + b)
2
4
⇔ 4(a + b) ≥ (a + b)
2
⇔ (a + b)(a + b −4) ≤ 0 ⇒ 0 ≤ a + b ≤ 4
Suy ra
P ≤ 16
Dấu bằng xảy ra khi a = b hay x = y =
1
2
.
Bài toán được chứng minh xong. 
Bài 18. Cho a, b, c đôi một khác nhau và thuộc [0; 2]. Tìm giá trị nhỏ nhất của
P =
1
(a −b)
2
+
1
(b −c)
2
+
1
(c −a)
2
Lời giải.
Áp dụng BĐT
1
x

2
+
1
y
2

8
(x + y)
2
với x, y > 0
Chứng minh điều này đơn giản vì nó tương đương với (x −y)
2
(x
2
+ y
2
+ 4xy) ≥ 0
Ta có
P ≥
8
(c −a)
2
+
1
(c −a)
2
=
9
(c −a)
2


9
4
Dấu "=" chẳng hạn khi a = 0, b = 1, c = 2.
Bài toán được chứng minh xong. 
Bài 19. Cho a, b, c là các số thực dương. Chứng minh rằng
a
2
2a
2
+ bc
+
b
2
2b
2
+ ac
+
c
2
2c
2
+ ab
≤ 1
Lời giải.
50 Bai toan bat dang thuc
Trang 20
Cách 1 . Bất đẳng thức đã cho tương đương với
bc
2(2a

2
+ bc)
+
ac
2(2b
2
+ ac)
+
ab
2(2c
2
+ ab)

1
2
Chỉ cần thêm bớt vào hai vế Sử dụng bất đẳng thức bunhiakowsky ta có
(bc)
2
2(2a
2
bc + (bc)
2
)
+
(ac)
2
2(2b
2
ac + (ac)
2

)
+
(ab)
2
2(2c
2
ab + (ab)
2
)

(bc + ca + ab)
2
2(ab + bc + ca)
2
=
1
2
Bài toán được chứng minh xong.
Cách 2 . Bất đẳng thức đã cho tương đương với
1
2 +
bc
a
2
+
1
2 +
ca
b
2

+
1
2 +
ab
c
2
≤ 1
Đặt x
2
=
bc
a
2
, y
2
=
ca
b
2
, z
2
=
ab
c
2
với x, y, z > 0, ta có xyz = 1
Bất đẳng thức cần chứng minh trở thành:
x
2
2 + x

2
+
y
2
2 + y
2
+
z
2
2 + z
2
≥ 1
Mặt khác, theo bất đẳng thức Cauchy − Schwarz, ta có
x
2
2 + x
2
+
y
2
2 + y
2
+
z
2
2 + z
2

(x + y + z)
2

6 + x
2
+ y
2
+ z
2
Ta cần chỉ ra
(x + y + z)
2
6 + x
2
+ y
2
+ z
2
≥ 1
nữa là bài toán được giải quyết. Thật vậy, bất đẳng thức trên tương đương với
xy + yz + zx ≥ 3
Bất đẳng cuối luôn đúng theo bất đẳng thức AM −GM.
Bài toán được chứng minh xong.
Cách 3 . Để ý rằng, theo bất đẳng thức AM-GM dạng 2xy ≤ x
2
+ y
2
, ta có
2a
2
2a
2
+ bc

=
1
1 +
bc
2a
2
=
1
1 +
1
2.
a
b
.
a
c
.

1
1 +
1
a
2
b
2
+
a
2
c
2

=
a
2
b
2
+ c
2
a
2
a
2
b
2
+ b
2
c
2
+ c
2
a
2
Thiết lập hai bất đẳng thức tương tự và cộng lại, ta thu được
2a
2
2a
2
+ bc
+
2b
2

2b
2
+ ca
+
2c
2
2c
2
+ ab

2(a
2
b
2
+ b
2
c
2
+ c
2
a
2
)
a
2
b
2
+ b
2
c

2
+ c
2
a
2
= 2
Hay
a
2
2a
2
+ bc
+
b
2
2b
2
+ ca
+
c
2
2c
2
+ ab
≤ 1
Bài toán được chứng minh xong.
50 Bai toan bat dang thuc
Trang 21
Bài 20. Cho x, y ∈ R thỏa mãn 5x
2

+ 5y
2
− 5x −15y + 8 ≤ 0. Tìm giá trị nhỏ nhất của
S = x + 3y
Lời giải.
Nhận xét. Đối với những bài toán cho giả thiết ax
2
+ bxy + cy
2
+ dx + ey + f = 0 (hay
≤ 0) và yêu cầu tìm giá trị lớn nhất, giá trị nhỏ nhất của biểu thức S = αx + βy (trong đó
a, b, c, d, e, f, α, β là các số đã cho trước rồi, lưu ý a > 0, b > 0) thì có một cách là ta rút x theo
S và y rồi thay vào biểu thức của giả thiết để được một tam thức bậc hai đối với ẩn y sau đó
giải điều kiện ∆
y
≥ 0 để thu được giá trị nhỏ nhất và giá trị lớn nhất của S.
Cụ thể bài này ta giải như sau
Do S = x + 3y nên x = S −3y, thay vào giả thiết 5x
2
+ 5y
2
−5x −15y + 8 ≤ 0 và viết theo hệ
số của biến y ta thu được
50y
2
− 30Sy + 5S
2
− 5S + 8 ≤ 0 (∗)
Vì bất đẳng thức trên đúng với mọi y nên ta có ∆ ≥ 0, tức là
900S

2
− 4.50.(5S
2
− 5S + 8) ≥ 0
Biến đổi tương đương ta thu được
−100S
2
+ 1000S − 1600 ≤ 0
Hay
100S
2
− 1000S + 1600 ≤ 0
Vì phương trình 100x
2
− 1000x + 1600 = 0 có 2 nghiệm là x = 2 và x = 8. Do đó ta thu được
2 ≤ S ≤ 8
Ta có:
Khi S = 2 thay vào (*) được 50y
2
− 60y + 18 ≤ 0 hay y =
3
5
nên x = S − 3y = 2 −
9
5
=
1
5
.
Khi S = 8 thay vào (*) được 50y

2
−240y + 288 ≤ 0 hay y =
12
5
nên x = S − 3y = 8 −
36
5
=
4
5
.
Vậy giá trị nhỏ nhất và giá trị lớn nhất của biểu thức S lần lượt là 2 và 8.
Bài toán được chứng minh xong.
Bài 21. Cho a, b, c > 0 thỏa mãn abc + a + c = b. Tìm giá trị lớn nhất của
P =
2
a
2
+ 1

2
b
2
+ 1
+
3
c
2
+ 1
Lời giải.

Ta có ac +
a
b
+
c
b
= 1.
Đặt tan
A
2
= a, tan
B
2
=
1
b
, tan
C
2
= c với A, B, C là ba góc của một tam giác.
Khi đó ta xét bài toán tổng quát sau với x, y, z là các số thực bất kì:
P =
x
a
2
+ 1

y
b
2

+ 1
+
z
c
2
+ 1
= x cos
2
A
2
− y sin
2
B
2
+ z cos
2
C
2
Vậy
P =

x
2

y
2
+
z
2


+
x
2
cos A +
y
2
cos B +
z
2
cos C
50 Bai toan bat dang thuc
Trang 22
Chú ý rằng chúng ta có một bất đẳng thức khá quen thuộc sau với m, n, p là các số thực:
2mn cos A + 2np cos B + 2pm cos C ≤ m
2
+ n
2
+ p
2
Khi đó để giải những bài như thế này ta định m, n, p sao cho
2mn =
x
2
, 2np =
y
2
, 2pm =
z
2
Vậy điều kiện của x, y, z là xyz phải dương. Khi đó ta có

P ≤
x
2

y
2
+
z
2
+ m
2
+ n
2
+ p
2
Ví dụ như bài ở trên ta có
mn =
x
4
=
1
2
, np =
y
4
=
1
2
, pm =
3

4
Khi đó ta có: m =

3
2
, n =

3
3
, p =

3
2
.Lúc này
P ≤
x
2

y
2
+
z
2
+ m
2
+ n
2
+ p
2
=

10
3
Đẳng thức xảy ra khi
Bài toán được chứng minh xong. 
Bài 22. Cho các số x, y, z thỏa mãn 1 ≤ x, y, z ≤ 4 và x ≥ y, x ≥ z. Tìm GTNN của biểu thức
P =
x
2x + 3y
+
y
y + z
+
z
x + z
Lời giải.
Cách 1 . Đặt
f(x, y, z) =
x
2x + 3y
+
y
y + z
+
z
x + z
Ta có
f(x, y,

xy) =
x

2x + 3y
+
2

y

x +

y
Xét hiệu
f(x, y, z) −f(x, y,

xy) =
(

xy − z)
2
(

x −

y)
(x + z)(y + z)(

x +

y)
≥ 0
Hiển nhiên đúng.
Suy ra

P = f(x, y, z) ≥
x
2x + 3y
+
2

y

x +

y
=
x
y
2
x
y
+ 3
+
2

x
y
+ 1
Tới đây đặt t =

x
y
với 1 ≤ t ≤ 2. Khi đó
P ≥

t
2
2t
2
+ 3
+
2
1 + t
Khảo sát hàm số f(t) =
t
2
2t
2
+ 3
+
2
1 + t
trên đoạn [1, 2] ta có f (t) ≥
34
33
dấu bằng xảy ra khi
t = 2 ⇔ x = 4y.
Hay x = 4, y = 1, z = 2.
Bài toán được chứng minh xong.
50 Bai toan bat dang thuc
Trang 23
Cách 2 . Ta có bổ đề sau:
1
1 + a
2

+
1
1 + b
2

1
1 + ab
với ab ≥ 1
Thật vậy, trừ vế theo vế, ta có ngay:
(a −b)
2
(ab −1)
(1 + a
2
)(1 + b
2
)(1 + ab)
≥ 0
Hiển nhiên đúng.

x
y
≥ 1, khi đó ta có:
P =
x
2x + 3y
+
y
y + z
+

z
x + z
=
x
y
2
x
y
+ 3
+
1
1 +
z
y
+
1
1 +
x
z

x
y
2
x
y
+ 3
+
2
1 +


x
y
Ta đưa được về dạng như trên.
Bài toán được chứng minh xong.
Cách 3 . Đặt
y
x
= a;
z
y
= b;
x
z
= c.
Khi đó abc = 1 và 2 ≥

bc ≥ 1.
Ta có
P =
1
2 + 3a
+
1
1 + b
+
1
1 + c
Xét bài toán mới này có các biến b và c bình đẳng nên ta dự đoán đẳng thức xảy ra khi
b = c =
1


a
.
Khi đó P =
1
2 + 3a
+
2

a
1 +

a
:= f(a) với a ∈

1
4
; 1

.
So sánh f

1
4

với f(1) ta dự đoán được P đạt giá trị nhỏ nhất khi a =
1
4
.
Khi đó b = c = 2 và ta tìm được các giá trị của (x, y, z) tương ứng là (4, 1, 2) .

Bài toán được chứng minh xong.
Bài 23. Cho x ≥ y ≥ z > 0 . Chứng minh rằng
x
2
y
z
+
y
2
z
x
+
z
2
x
y
≥ x
2
+ y
2
+ z
2
Lời giải.
Cách 1 . Áp dụng BĐT Bunhiacopsky cho 2 dãy số x

y
z
; y

z

x
; z

x
y
và x

z
y
; y

x
z
; z

y
x
Ta có:
(
x
2
y
z
+
y
2
z
x
+
z

2
x
y
)(
x
2
z
y
+
y
2
x
z
+
z
2
y
x
) ≥ (x
2
+ y
2
+ z
2
)
2
(1)
Xét hiệu:
F =
x

2
y
z
+
y
2
z
x
+
z
2
x
y
− (
x
2
z
y
+
y
2
x
z
+
z
2
y
x
)
=

1
xyz
(x
3
z
2
+ y
3
z
2
+ z
3
x
2
− x
3
z
2
− y
3
z
2
− z
3
y
2
)
=
1
xyz

[(x
3
z
2
− y
3
z
2
) + (y
3
z
2
− z
3
y
2
) + (z
3
x
2
− x
3
z
2
)]
=
1
xyz
(x −y)(y − z)(x −z)(xy + yz + zx) ≥ 0
50 Bai toan bat dang thuc

Trang 24
Suy ra:
x
2
y
z
+
y
2
z
x
+
z
2
x
y

x
2
z
y
+
y
2
x
z
+
z
2
y

x
(2)
Từ (1) và (2), ta được:
(
x
2
y
z
+
y
2
z
x
+
z
2
x
y
)
2
≥ (
x
2
y
z
+
y
2
z
x

+
z
2
x
y
)(
x
2
z
y
+
y
2
x
z
+
z
2
y
x
) ≥ (x
2
+ y
2
+ z
2
)
2
Vậy:
(

x
2
y
z
+
y
2
z
x
+
z
2
x
y
)
2
≥ (x
2
+ y
2
+ z
2
)
2

x
2
y
z
+

y
2
z
x
+
z
2
x
y
≥ x
2
+ y
2
+ z
2
Đẳng thức xảy ra khi và chỉ khi x = y = z > 0.
Bài toán được chứng minh xong.
Cách 2 . Bất đẳng thức cần chứng minh tương đương với
x
2
y
z
+
y
2
z
x
+
z
2

x
y
+ zx ≥ x
2
+ y
2
+ zx + z
2
Vì sao lại thêm bớt zx vào hai vế? Vì với một bài toán hoán vị, trong đó y là số nằm giữa x và
z thì thường ta sẽ, làm thế nào đó, để sử dụng được đánh giá quen thuộc y
2
+ zx ≤ y(z + x).
Thật vậy, nếu sử dụng đánh giá trên thì bài toán quy về chứng minh
x
2
y
z
+
y
2
z
x
+
z
2
x
y
+ zx ≥ x
2
+ z

2
+ xy + yz
Một cách tự nhiên ta muốn sử dụng bất đẳng thức AM-GM dạng x + y ≥ 2

xy để có
y
2
z
x
+ zx ≥ 2

y
2
z
x
.zx = 2yz
Từ đó, ta sẽ thử chứng minh bất đẳng thức mạnh hơn là
x
2
y
z
+ 2yz +
z
2
x
y
≥ x
2
+ z
2

+ xy + yz
Hay
x
2
y
z
+ yz +
z
2
x
y
≥ x
2
+ z
2
+ xy
Tương đương với

x
2
y
z
− x
2

+

yz − z
2


+

z
2
x
y
− xy

≥ 0
Hay
(y − z)

x
2
z
+ z −
x(y + z)
y

≥ 0
Bất đẳng thức cuối này hiển nhiên đúng vì y ≥ z và
x
2
z
+ z ≥ 2x ≥
x(y + z)
y
Bài toán được chứng minh xong.
50 Bai toan bat dang thuc
Trang 25

×